Easy divergence theorem problem

Click For Summary
The discussion focuses on evaluating the flux integral using the Divergence Theorem for the vector field F(x,y,z)=2xi+3yj+4zk over the sphere defined by x^2+y^2+z^2=9, with the expected answer being 324pi. A participant calculated the divergence and set up a triple integral but expressed uncertainty about using polar coordinates and setting the bounds. Another contributor suggested using spherical coordinates, providing the differential volume element and appropriate limits for integration. The conversation emphasizes the importance of understanding the integration process rather than relying solely on formulas. Overall, the discussion highlights the application of the Divergence Theorem and the use of spherical coordinates for solving the problem.
xmflea
Messages
43
Reaction score
0
Evaluate the flux integral using the Divergence Theorem if F(x,y,z)=2xi+3yj+4zk
and S is the sphere x^2+y^2+z^2=9
answer is 324pi

so far i took the partial derivitavs of i j k for x y z and added them to get 9.

so i have the triple integral of 9 dzdxdy

i think u have to use polar cordinates, but I am reeally bad at those. and do not know how to set the bounds for this problem. can someone help me get started with that? thanks.
 
Physics news on Phys.org
see what i could do...is since i know volume of a sphere is 4/3r^3pi, i can just say that the radius is 3, and get 36pi x 9. to get 324pi. but I am sure my teacher would expect me to do some actual integration.
 
If I were your teacher, I wouldn't! Intelligence trumps blindly applying formulas any day.

If you really do want to integrate, use spherical coordinates. You book probably gives an example of using spherical coordinates to find the volume of a sphere. The "differential of volume" in spherical coordinates is \rho^2 sin(\phi) d\rho d\theta\dphi and the limits of integration are \rho from 0 to 3, \theta from 0 to 2\pi and \phi from 0 to \phi.
 
Question: A clock's minute hand has length 4 and its hour hand has length 3. What is the distance between the tips at the moment when it is increasing most rapidly?(Putnam Exam Question) Answer: Making assumption that both the hands moves at constant angular velocities, the answer is ## \sqrt{7} .## But don't you think this assumption is somewhat doubtful and wrong?

Similar threads

  • · Replies 6 ·
Replies
6
Views
2K
Replies
8
Views
3K
  • · Replies 1 ·
Replies
1
Views
1K
  • · Replies 3 ·
Replies
3
Views
2K
Replies
3
Views
2K
  • · Replies 3 ·
Replies
3
Views
2K
  • · Replies 2 ·
Replies
2
Views
2K
  • · Replies 20 ·
Replies
20
Views
4K
  • · Replies 6 ·
Replies
6
Views
2K
Replies
2
Views
2K